Maternity final

Pataasin ang iyong marka sa homework at exams ngayon gamit ang Quizwiz!

An antenatal client is informing the nurse of her prenatal signs and symptoms. Which of the following findings would the nurse determine are presumptive signs of pregnancy? Select all that apply. 1. Amenorrhea. 2. Breast tenderness. 3. Quickening. 4. Frequent urination. 5. Uterine growth.

1, 2, 3, and 4 are correct. 1. Amenorrhea is a presumptive sign of pregnancy. 2. Breast tenderness is a presumptive sign of pregnancy. 3. Quickening is a presumptive sign of pregnancy. 4. Frequent urination is a presumptive sign of pregnancy.

A client enters the prenatal clinic. She states that she believes she is pregnant. Which of the following hormone elevations will indicate a high probability that the client is pregnant? 1. Chorionic gonadotropin. 2. Oxytocin. 3. Prolactin. 4. Luteinizing hormone.

1. Chorionic gonadotropin High levels of the hormone chorionic gonadotropin in the bloodstream and urine of the woman is a probable sign of pregnancy.

Peer victimization is becoming a significant problem for school-age children and adolescents in the United States. Parents should be educated regarding signs that a child is being bullied. These might include (select all that apply) a. The child spends an inordinate amount of time in the nurse's office. b. Belongings frequently go missing or are damaged. c. The child wants to be driven to school. d. School performance improves. e. The child freely talks about his day.

A . The child spends an inordinate amount of time in the nurse's office. B. Belongings frequently go missing or are damaged. C.The child wants to be driven to school. Feedback Correct Signs that may indicate a child is being bullied are similar to signs of other types of stress and include nonspecific illness or complaints, withdrawal, depression, school refusal, and decreased school performance. Children express fear of going to school or riding the school bus. Very often, children will not talk about what is happening to them. Incorrect These are not indications of bullying.

What information should be given to the parents of a 12-month-old child regarding appropriate play activities for this age? a.Give large push-pull toys for kinetic stimulation. b.Place a cradle gym across the crib to help develop fine motor skills. c.Provide the child with finger paints to enhance fine motor skills. d.Provide a stick horse to develop gross motor coordination.

A .Give large push-pull toys for kinetic stimulation. A 12-month-old child is able to pull to a stand and walk holding on or independently. Appropriate toys for this age child include large push-pull toys for kinetic stimulation. A cradle gym should not be placed across the crib. Finger paints are appropriate for older children. A 12-month-old child does not have the stability to use a stick horse.

Along with gas exchange and nutrient transfer, the placenta produces many hormones necessary for normal pregnancy. These include which of the following? Select all that apply. a.Human chorionic gonadotropin (hCG) b.Insulin c.Estrogen d.Progesterone e.Testosterone

A, C, D. hCG causes the corpus luteum to persist and produce the necessary estrogens and progesterone for the first 6 to 8 weeks. Estrogens cause enlargement of the woman's uterus and breasts; growth of the ductal system in the breasts; and, as term approaches, plays a role in the initiation of labor. Progesterone causes the endometrium to change, providing early nourishment. Progesterone also protects against spontaneous abortion by suppressing maternal reactions to fetal antigens and reduces unnecessary uterine contractions. Other hormones produced by the placenta include hCT, hCA, and a number of growth factors.

A nurse is caring for a client who is 1 day postpartum. The nurse is assessing for maternal adaptation and mother-infant bonding. Which of the following behaviors by the client indicates a need for the nurse to intervene? (Select all that apply.) A. Demonstrates apathy when the infant cries B. Touches the infant and maintains close physical proximity C. Views the infant's behavior as uncooperative during diaper changing D. Identifies and relates infant's characteristics to those of family members E. Interprets the infant's behavior as meaningful and a way of expressing need

A. Demonstrates apathy when the infant cries C. Views the infant's behavior as uncooperative during diaper changing Feedback: A.This behavior demonstrates a lack of interest in the infant and impaired maternal-infant bonding. C. A client's view of her infant as being uncooperative during diaper changing is a sign of impaired maternal-infant bonding.

Which statement best describes development in infants and children? a. Development, a predictable and orderly process, occurs at varying rates within normal limits. b. Development is primarily related to the growth in the number and size of cells. c. Development occurs in a proximodistal direction with fine muscle development occurring first. d. Development is more easily and accurately measured than growth.

A. Development, a predictable and orderly process, occurs at varying rates within normal limits.

The nurse working in the ER is assessing a 12 year old patient. It is noted that there are several bruises on the body. The child is very hesitant to speak about the injuries and his father states his son is very clumsy and takes too many risks. what should the nurse do at this point? a. Document his assessment in the chart as being clumsy 12 year old. b. Talk with the 12 year old and instruct him to be more careful in his play. c. Talk with the dad about protecting the child from harm. d. notify the CPS regarding the suspicion of abuse

ANS: D - notify the CPS regarding the suspicion of abuse

Sara, age 4 months, was born at 35 weeks' gestation. She seems to be developing normally, but her parents are concerned because she is a "more difficult" baby than their other child, who was term. The nurse should explain that: a. Infants' temperaments are part of their unique characteristics. b. Infants become less difficult if they are not kept on scheduled feedings and structured routines. c. Sara's behavior is suggestive of failure to bond completely with her parents. d. Sara's difficult temperament is the result of painful experiences in the neonatal period.

A. infants' temperaments are part of their unique characteristics Infant temperament has a strong biologic component. Together with interactions with the environment, primarily the family, the biologic component contributes to the infant's unique temperament. Children perceived as difficult may respond better to scheduled feedings and structured caregiving routines than to demand feedings and frequent changes in routines. Sara's temperament has been created by both biologic and environmental factors. The nurse should provide guidance in parenting techniques that are best suited to Sara's temperament.

Both members of an expectant couple are carriers for phenylketonuria (PKU), an autosomal recessive disorder. In counseling them about the risk to their unborn child, the nurse should tell them that a. The child has a 25% chance of being affected. b. The child will be a carrier, like the parents. c. The child has a 50% chance of being affected. d. One of four of their children will be affected.

A. the child has a 25% chance of being affected

Rho immune globulin will be ordered postpartum if which situation occurs? a. Mother Rh-, baby Rh+ b. Mother Rh-, baby Rh- c. Mother Rh+, baby Rh+ d. Mother Rh+, baby Rh-

A.Mother Rh-, baby Rh+ An Rh- mother delivering an Rh+ baby may develop antibodies to fetal cells that entered her bloodstream when the placenta separated. The Rho immune globulin works to destroy the fetal cells in the maternal circulation before sensitization occurs. If mother and baby are both Rh+ or Rh- the blood types are alike, so no antibody formation would be anticipated. If the Rh+ blood of the mother comes in contact with the Rh- blood of the infant, no antibodies would develop because the antigens are in the mother's blood, not the infant's.

Teasing can be common during the school-age years. The nurse should recognize that which applies to teasing? a. Can have a lasting effect on children b. Is not a significant threat to self-concept c. Is rarely based on anything that is concrete d. Is usually ignored by the child who is being teased

ANS: a. Can have a lasting effect on children Teasing in this age group is common and can have a long-lasting effect. Increasing awareness of differences, especially when accompanied by unkind comments and taunts from others, may make a child feel inferior and undesirable. Physical impairments such as hearing or visual defects, ears that "stick out," or birth marks assume great importance.

The nurse is doing a routine assessment on a 18-month-old infant and notes that the anterior fontanel is closed. This should be interpreted as: a. A normal finding. b. A questionable finding—the infant should be rechecked in 1 month. c. An abnormal finding—indicates the need for immediate referral to a practitioner. d. An abnormal finding—indicates the need for developmental assessment.

ANS: A. a normal finding Because the anterior fontanel normally closes between ages 12 and 18 months, this is a normal finding, and no further intervention is required.

A 3-month-old infant, born at 38 weeks of gestation, will hold a rattle if it is put in her hands; however, she will not voluntarily grasp it. The nurse should interpret this as: a. Normal development. b. Significant developmental lag. c. Slightly delayed development caused by prematurity. d. Suggestive of a neurologic disorder such as cerebral palsy.

ANS: A. normal development This indicates normal development. Reflexive grasping occurs during the first 2 to 3 months and then gradually becomes voluntary. No evidence of developmental lag, delayed development, or neurologic dysfunction is present.

When planning care for adolescents, the nurse should a. Teach parents first, and they, in turn, will teach the teenager. b. Provide information for their long-term health needs because teenagers respond best to long-range planning. c. Maintain the parents' role by providing explanations for treatment and procedures to the parents only. d. Give information privately to adolescents about how they can manage the specific problems that they identify.

ANS: D. Give information privately to adolescents about how they can manage the specific problems that they identify. Feedback A Teenagers are socially and cognitively at the developmental stage where the health care provider can teach them. B Teenagers are more interested in immediate health care needs than in long-term needs. C Teenagers are at the developmental level that allows them to receive explanations about health care directly from the nurse. D Problems that teenagers identify and are interested in are typically the problems that they are the most willing to address. Confidentiality is important to adolescents. Adolescents prefer to confer privately (without parents) with the nurse and health care provider.

A nurse is caring for a 4 year old what are the best foods to ensure adequate calcium intake. Select all that apply: a. soft serve yogurt b. orange juice c. chocolate milk d. green beans e. swiss cheese

ANS: a. soft serve yogurt c. chocolate milk e. swiss cheese

A nurse in the newborn nursery is monitoring a preterm newborn infant for respiratory distress syndrome. Which assessment signs if noted in the newborn infant would alert the nurse to the possibility of this syndrome? 1. Hypotension and Bradycardia 2. Tachypnea and retractions 3. Acrocyanosis and grunting 4. The presence of a barrel chest with grunting

ANS: 2. Tachypnea and retractions Feedback: The infant with respiratory distress syndrome may present with signs of cyanosis, tachypnea or apnea, nasal flaring, chest wall retractions, or audible grunts.

What type of family is one in which all members are related by blood? a. Consanguineous b. Affinal c. Family of origin d. Household

ANS: A - Consanguineous A consanguineous family is one of the most common types and consists of members who have a blood relationship. The affinal family is one made up of marital relationships. Although the parents are married, they may each bring children from a previous relationship. The family of origin is the family unit that a person is born into. Considerable controversy has been generated about the newer concepts of families (i.e., communal, single-parent or homosexual families). To accommodate these other varieties of family styles, the descriptive term household is frequently used.

The nurse is obtaining vital signs on a 1-year-old child. What is the most appropriate site for assessing the pulse rate? a. Apical b. Radial c. Carotid d. Femoral

ANS: A -Apical Feedback A Apical pulse rates are taken in children younger than 2 years. B Radial pulse rates may be taken in children older than 2 years. C It is difficult to palpate the carotid pulse in an infant. D The femoral pulse is palpated when comparing peripheral pulses, but it is not used to measure an infant's pulse rate.

The nurse providing newborn stabilization must be aware that the primary side effect of maternal narcotic analgesia in the newborn is a. Respiratory depression b. Bradycardia c. Acrocyanosis d. Tachypnea

ANS: A Respiratory depression Feedback A. An infant delivered within 1 to 4 hours of maternal analgesic administration is at risk for respiratory depression from the sedative effects of the narcotic. B Bradycardia is not the anticipated side effect of maternal analgesics. C Acrocyanosis is an expected finding in a newborn and is not related to maternal analgesics. D The infant who is having a side effect to maternal analgesics normally would have a decrease in respirations, not an increase.

Part of the health assessment of a newborn is observing the infant's breathing pattern. A full-term newborn's breathing pattern is predominantly: a.Abdominal with synchronous chest movements. b.Chest breathing with nasal flaring. c.Diaphragmatic with chest retraction. d.Deep with a regular rhythm.

ANS: A. Abdominal with synchronous chest movements. Feedback: In normal infant respiration the chest and abdomen rise synchronously, and breaths are shallow and irregular. Breathing with nasal flaring is a sign of respiratory distress. Diaphragmatic breathing with chest retraction is a sign of respiratory distress. Infant breaths are not deep with a regular rhythm.

The nurse has just started a new shift and is reviewing the chart for her assigned patient. The patient is 6 cm dilated, 100% effaced, -4 station with intact membranes. Ten minutes later, the patient informs the nurse that her membranes have just ruptured. The nurse notices variable decelerations on the monitor. The nurse's next action should be to A. Assess for a prolapsed cord B. Increase the intravenous fluids and start oxygen C. Notify the nurse-midwife D. Nothing, this is normal immediately after membranes rupture

ANS: A. Assess for a prolapsed cord A. With a -4 station, the fetus is at high risk for a prolapsed cord when the membranes rupture. B. If the pattern had been caused by uteroplacental insufficiency, then increasing the intravenous fluids and starting oxygen would be appropriate. C. It is important to notify the primary care giver with this pattern; however, it is not the first priority. The nurse should assess for a prolapsed cord and try to relieve the pressure. D. This is not a normal pattern after membranes rupture. It is nonreassuring.

The placenta allows exchange of oxygen, nutrients, and waste products between the mother and fetus by a. Contact between maternal blood and fetal capillaries within the chorionic villi b. Interaction of maternal and fetal pH levels within the endometrial vessels c. A mixture of maternal and fetal blood within the intervillous spaces d. Passive diffusion of maternal carbon dioxide and oxygen into the fetal capillaries

ANS: A. Contact between maternal blood and fetal capillaries within the chorionic villi Feedback A Fetal capillaries within the chorionic villi are bathed with oxygen- and nutrient-rich maternal blood within th

Nurses can prevent evaporative heat loss in the newborn by: a. Drying the baby after birth and wrapping the baby in a dry blanket. b. Keeping the baby out of drafts and away from air conditioners. c. Placing the baby away from the outside wall and the windows. d. Warming the stethoscope and nurse's hands before touching the baby.

ANS: A. Drying the baby after birth and wrapping the baby in a dry blanket. Feedback: Wet linens or wet clothes can cause heat loss by evaporation. If the heat loss is caused by placing the baby near cold surfaces or equipment, it is termed a radiation heat loss. Heat loss by convection occurs when drafts come from open doors and air currents created by people moving around. Conduction heat loss occurs when the baby comes into contact with cold objects or surfaces.

Which mechanism of labor occurs when the largest diameter of the fetal presenting part passes the pelvic inlet? a. Engagement b. Extension c. Internal rotation d. External rotation

ANS: A. ENGAGEMENT Engagement occurs when the presenting part fully enters the pelvic inlet. Extension occurs when the fetal head meets resistance from the tissues of the pelvic floor and the fetal neck stops under the symphysis. This causes the fetal head to extend. Internal rotation occurs when the fetus enters the pelvic inlet. The rotation allows the longest fetal head diameter to conform to the longest diameter of the maternal pelvis. External rotation occurs after the birth of the head. The head then turns to the side so the shoulders can internally rotate and are positioned with their transverse diameter in the anteroposterior diameter of the pelvic outlet.

The nurse is using the Ballard scale to determine the gestational age of a newborn. Which assessment finding is consistent with a gestational age of 40 weeks? A. Flexed posture B. Abundant lanugo C. Smooth, pink skin with visible veins D. Faint red marks on the soles of the feet

ANS: A. FLEXED POSTURE Feedback: A. Correct: Term infants typically have a flexed posture. B. Incorrect: Abundant lanugo usually is seen on preterm infants. C. Incorrect: Smooth, pink skin with visible veins is seen on preterm infants. D. Incorrect: Faint red marks usually are seen on a preterm infant.

A new father states, "I know nothing about babies," but he seems to be interested in learning. The nurse should take which action? a. Include him in teaching sessions. b. Tell him when he does something wrong. c. Show no concern because he will learn on his own. d. Continue to observe his interaction with the newborn.

ANS: A. Include him in teaching sessions. The nurse must be sensitive to the father's needs and include him whenever possible. He should be encouraged by pointing out the correct procedures he does. By criticizing, he will be discouraged. Showing no concern is not a nursing role. Nurses need to be sensitive to clients' needs. It is important to note the bonding process of the mother and the father, but that does not satisfy the expressed needs of the father.

A pregnant woman wants to breastfeed her infant; however, her husband is not convinced that there are any scientific reasons to do so. The nurse can give the couple printed information comparing breastfeeding and bottle-feeding. Which statement is most accurate? Bottle-feeding using commercially prepared infant formulas: a.Increases the risk that the infant will develop allergies. b.Helps the infant sleep through the night. c.Ensures that the infant is getting iron in a form that is easily absorbed. d.Requires that multivitamin supplements be given to the infant.

ANS: A. Increases the risk that the infant will develop allergies. Feedback: Exposure to cow's milk poses a risk of developing allergies, eczema, and asthma. "Bottle-feeding using commercially prepared infant formulas helps the infant sleep through the night" is a false statement. Iron is better absorbed from breast milk than from formula. Commercial formulas are designed to meet the nutritional needs of the infant and resemble breast milk.

The best time to teach nonpharmacologic pain control methods to an unprepared laboring woman is during which phase? a. Latent phase b. Active phase c. Transition phase d. Second stage

ANS: A. Latent phase Feedback A The latent phase of labor is the best time for intrapartum teaching, because the woman is usually anxious enough to be attentive, yet comfortable enough to understand the teaching. B During the active phase, the woman is focused internally and unable to concentrate on teaching. C During transition, the woman is focused on keeping control; she is unable to focus on anyone else or learn at this time. D During the second stage, the woman is focused on pushing. She normally handles the pain better at this point because she is active in doing something to hasten the delivery.

Excessive anxiety in labor heightens the woman's sensitivity to pain by increasing a. Muscle tension b. Blood flow to the uterus c. The pain threshold d. Rest time between contractions

ANS: A. MUSCLE TENSION Feedback A Anxiety and fear increase muscle tension, diverting oxygenated blood to the woman's brain and skeletal muscles. Prolonged tension results in general fatigue, increased pain perception, and reduced ability to use coping skills. B Anxiety can decrease blood flow to the uterus. C Anxiety will decrease the pain threshold. D Anxiety will decrease the amount of rest the mother gets between contractions.

Which hormone is necessary for milk production? a. Prolactin b. Progesterone c. Lactogen d. Estrogen

ANS: A. Prolactine

Which part of the mature sperm contains the male chromosomes? a.The head of the sperm b.The middle portion of the sperm c.X-bearing sperm d.The tail of the sperm

ANS: A. The head of the sperm The head of the sperm contains the male chromosomes that will join the chromosomes of the ovum. The middle portion of the sperm supplies energy for the tail's whip-like action. If an X-bearing sperm fertilizes the ovum, the baby will be female. The tail of the sperm helps propel the sperm toward the ovum.

A new mother asks the nurse about the "white substance" covering her infant. How should the nurse explain the purpose of vernix caseosa? a. Vernix caseosa protects the fetal skin from the amniotic fluid. b. Vernix caseosa promotes the normal development of the peripheral nervous system. c. Vernix caseosa allows the transport of oxygen and nutrients across the amnion. d. Vernix caseosa regulates fetal temperature.

ANS: A. Vernix caseosa protects the fetal skin from the amniotic fluid. feedback: Prolonged exposure to the amniotic fluid during the fetal period could result in the breakdown of the skin without the protection of the vernix caseosa. Normal development of the peripheral nervous system was dependent on nutritional intake of the mother. The amnion was the inner membrane that surrounded the fetus and was not involved in the oxygen and nutrient exchange. The amniotic fluid helped maintain fetal temperature.

During labor a vaginal examination should be performed only when necessary because of the risk of: a. infection. b. fetal injury. c. discomfort. d. perineal trauma.

ANS: A. infection Vaginal examinations increase the risk of infection by carrying vaginal microorganisms upward toward the uterus. Properly performed vaginal examinations should not cause fetal injury. Vaginal examinations may be uncomfortable for some women in labor, but that is not the main reason for limiting them. A properly performed vaginal examination should not cause perineal trauma.

New parents express concern that, because of the mother's emergency cesarean birth under general anesthesia, they did not have the opportunity to hold and bond with their daughter immediately after her birth. The nurse's response should convey to the parents that: a. Attachment, or bonding, is a process that occurs over time and does not require early contact. b. The time immediately after birth is a critical period for people. c. Early contact is essential for optimum parent-infant relationships. d. They should just be happy that the infant is healthy.

ANS: A.Attachment, or bonding, is a process that occurs over time and does not require early contact. Feedback: Attachment, or bonding, is a process that occurs over time and does not require early contact. The formerly accepted definition of bonding held that the period immediately after birth was a critical time for bonding to occur. Research since has indicated that parent-infant attachment occurs over time. A delay does not inhibit the process. Parent-infant attachment involves activities such as touching, holding, and gazing; it is not exclusively eye contact. A response that conveys that the parents should just be happy that the infant is healthy is inappropriate because it is derogatory and belittling

The nurse providing couplet care should understand that nipple confusion results when: a. Breastfeeding babies receive supplementary bottle feedings. b. The baby is weaned too abruptly. c. Pacifiers are used before breastfeeding is established. d. Twins are breastfed together.

ANS: A.Breastfeeding babies receive supplementary bottle feedings

If the client's white blood cell (WBC) count is 25,000/mm3 on her second postpartum day, which action should the nurse take? a. Document the finding. b. Tell the health care provider. c. Begin antibiotic therapy immediately. d. Have the laboratory draw blood for reanalysis.

ANS: A.Document the finding. An increase in WBC count to 25,000/mm3 during the postpartum period is considered normal and not a sign of infection. The nurse should document the finding. Because this is a normal finding, there is no reason to alert the health care provider. Antibiotics are not needed because the elevated WBCs are caused by the stress of labor and not an infectious process. There is no need for reassessment as it is expected for the WBCs to be elevated.

Which statement is most descriptive of central nervous system stimulants? a.They produce strong physical dependence. b.They can result in strong psychologic dependence. c.Withdrawal symptoms are life threatening. d.Acute intoxication can lead to coma

ANS: B Central nervous system stimulants such as amphetamines and cocaine produce a strong psychologic dependence. This class of drugs does not produce strong physical dependence and can be withdrawn without much danger. Acute intoxication leads to violent, aggressive behavior or psychotic episodes characterized by paranoia, uncontrollable agitation, and restlessness

Which statement is the most appropriate advice to give parents of a 16-year-old who is rebellious? a. "You need to be stricter so that your teen stops trying to test the limits." b. "You need to collaborate with your daughter and set limits that are perceived as being reasonable." c. "Increasing your teen's involvement with her peers will improve her self-esteem." d. "Allow your teenager to choose the type of discipline that is used in your home."

ANS: B "You need to collaborate with your daughter and set limits that are perceived as being reasonable." Feedback A Setting stricter limits typically does not decrease rebelliousness or decrease testing of parental limits. B Allowing teenagers to choose between realistic options and offering consistent and structured discipline typically enhances cooperation and decreases rebelliousness. Structure helps adolescents to feel more secure and assists them in the decision-making process. C Increasing peer involvement does not typically increase self-esteem. D Allowing teenagers to choose the method of discipline is not realistic and typically does not reduce rebelliousness.

The mother of a 14-month-old child is concerned because the child's appetite has decreased. The best response for the nurse to make to the mother is: a. "It is important for your toddler to eat three meals a day and nothing in between." b. "It is not unusual for toddlers to eat less." c. "Be sure to increase your child's milk consumption, which will improve nutrition." d. "Giving your child a multivitamin supplement daily will increase your toddler's appetite."

ANS: B - "It is not unusual for toddlers to eat less." A Toddlers need small, frequent meals. Nutritious selection throughout the day, rather than quantity, is more important with this age-group. B Physiologically, growth slows and appetite decreases during the toddler period. C Milk consumption should not exceed 16 to 24 oz daily. Juice should be limited to 4 to 6 oz per day. Increasing the amount of milk will only further decrease solid food intake. D Supplemental vitamins are important for all children, but they do not increase appetite.

A school nurse is conducting a class with adolescents on suicide. Which true statement about suicide should the nurse include in the teaching session? a. A sense of hopelessness and despair are a normal part of adolescence. b. Gay and lesbian adolescents are at a particularly high risk for suicide. c. Problem-solving skills are of limited value to the suicidal adolescent. d. Previous suicide attempts are not an indication of risk for completed suicides.

ANS: B - Gay and lesbian adolescents are at a particularly high risk for suicide. A significant number of teenage suicides occur among homosexual youths. Gay and lesbian adolescents who live in families or communities that do not accept homosexuality are likely to suffer low self-esteem, self-loathing, depression, and hopelessness as a result of a lack of acceptance from their family or community. At-risk teenagers include those who are depressed, have poor problem-solving skills, or use drugs and alcohol. History of previous suicide attempt is a serious indicator for possible suicide completion in the future.

An important consideration for the school nurse who is planning a class on bicycle safety is: a. Most bicycle injuries involve collision with an automobile. b. Head injuries are the major causes of bicycle-related fatalities. c. Children should wear bicycle helmets if they ride on paved streets. d. Children should not ride double unless the bicycle has an extra-large seat.

ANS: B - Head injuries are the major causes of bicycle-related fatalities. The most important aspect of bicycle safety is to encourage the rider to use a protective helmet. Head injuries are the major cause of bicycle-related fatalities. Although motor vehicle collisions do cause injuries to bicyclists, most injuries result from falls. The child should always wear a properly fitted helmet approved by the U.S. Consumer Product Safety Commission. Children should not ride double.

The most important nursing intervention after the injection of epidural anesthesia is monitoring a. Urinary output b. contractions c. intravenous infusion rate d. maternal blood pressure

D. maternal blood pressure

A new father wants to know what medication was put into his infant's eyes and why it is needed. The nurse explains to the father that the purpose of the Ilotycin ophthalmic ointment is to: A. Destroy an infectious exudate caused by Staphylococcus that could make the infant blind B. Prevent gonorrheal and chlamydial infection of the infant's eyes potentially acquired from the birth canal C. Prevent potentially harmful exudate from invading the tear ducts of the infant's eyes, leading to dry eyes D. Prevent the infant's eyelids from sticking together and help the infant see

ANS: B - Prevent gonorrheal and chlamydial infection of the infant's eyes potentially acquired from the birth canal Feedback: A. Incorrect: Prophylactic ophthalmic ointment is instilled in the eyes of all neonates to prevent gonorrheal or chlamydial infection. B. Correct: This is an accurate explanation. C. Incorrect: Prophylactic ophthalmic ointment is not instilled to prevent dry eyes. It is instilled to prevent gonorrheal or chlamydial infection. D. Incorrect: Prophylactic ophthalmic ointment has no bearing on vision other than to protect against infection that may lead to vision problems.

A nurse is assessing an older school-age child recently admitted to the hospital. Which assessment indicates that the child is in an appropriate stage of cognitive development? a. The child's addition and subtraction ability b. The child's ability to classify c. The child's vocabulary d. The child's play activity

ANS: B - The child's ability to classify Feedback A Subtraction and addition are appropriate cognitive activities for the young school-age child. B The ability to classify things from simple to complex and the ability to identify differences and similarities are cognitive skills of the older school-age child; this demonstrates use of classification and logical thought processes. C Vocabulary is not as valid an assessment of cognitive ability as is the child's ability to classify. D Play activity is not as valid an assessment of cognitive function as is the child's ability to classify.

The nurse sees a pattern on the fetal monitor that looks similar to early decelerations, but the deceleration begins near the acme of the contraction and continues well beyond the end of the contraction. Which nursing action indicates the proper evaluation of this situation? a. Continue to monitor these early decelerations, which occur as the fetal head is compressed during a contraction. b. This deceleration pattern is associated with uteroplacental insufficiency, so the nurse acts quickly to improve placental blood flow and fetal oxygen supply. c. This pattern reflects variable decelerations. No interventions are necessary at this time. d. Document this reassuring fetal heart rate pattern, but decrease the rate of the intravenous fluid.

ANS: B This deceleration pattern is associated with uteroplacental insufficiency, so the nurse acts quickly to improve placental blood flow and fetal oxygen supply. Feedback A These are late decelerations, not early; therefore interventions are necessary. B This is a description of a late deceleration. Oxygen should be given via snug facemask. Position the woman on her left side to increase placental blood flow. C Variable decelerations are caused by cord compression. A vaginal examination should be performed to identify this potential emergency. D This is not a reassuring pattern, so the intravenous rate should be increased to increase the mother's blood volume.

The nurse is caring for a postpartum patient. One day after delivery, the nurse assesses the lochia of the patient and finds that it is red and has a foul-smelling odor. What does the nurse conclude from this assessment? A. The patient is healing well. B. An infection is present. C. There is evidence of clinical dehydration. D. There is potential internal hemorrhaging.

ANS: B an infection is present Lochia is vaginal discharge that can be observed after childbirth; it contains blood and mucus. Usually, lochia appears to be dark red during the first 3 days after delivery and has fleshy odor. The presence of foul odor indicates that the patient has an infection. Foul-smelling lochia does not indicate that the patient is healing well. The presence of a moderate amount of lochia would indicate that the patient is healing well. An increase in body temperature would indicate that the patient is dehydrated. Excessive bleeding would indicate internal hemorrhage.

With regard to the structure and function of the placenta, the maternity nurse should be aware that: a.As the placenta widens, it gradually thins to allow easier passage of air and nutrients b.As one of its early functions, the placenta acts as an endocrine gland c.The placenta is able to keep out most potentially toxic substances, such as cigarette smoke, to which the mother is exposed d.Optimal blood circulation is achieved through the placenta when the woman is lying on her back or standing

ANS: B. As one of its early functions, the placenta acts as an endocrine gland The placenta produces four hormones necessary to maintain the pregnancy. The placenta widens until week 20 and continues to grow thicker. Toxic substances such as nicotine and carbon monoxide readily cross the placenta into the fetus. Optimal circulation occurs when the woman is lying on her side.

After a forceps-assisted birth, the mother is observed to have continuous bright red lochia but a firm fundus. What other data indicates the presence of a potential vaginal wall hematoma? a. Mild, intermittent perineal pain b. Edema and discoloration of the labia and perineum c. Lack of an episiotomy d. Lack of pain in the perineal area

ANS: B. Edema and discoloration of the labia and perineum Feedback A The pain with vaginal hematoma is severe and constant. B The nurse should monitor for edema and discoloration. Using a cold application to the labia and perineum reduces pain by numbing the area and limiting bruising and edema for the first 12 hours. C An episiotomy is performed as the fetal head distends the perineum. D The pain associated with vaginal hematoma is severe.

Which intervention may be most helpful for the patient experiencing most of her labor pain in her back? a. Lying supine with head slightly elevated b. Having the support person apply firm pressure to the sacrum c. Sitting upright with the legs straight d. Having the nurse massage the upper back during a contraction

ANS: B. Having the support person apply firm pressure to the sacrum Feedback A-This would put more pressure onto the lower back area and decrease placental profusion. B-Firm pressure against the sacrum may be helpful in relieving the discomfort associated with back labor. C-This position would put more pressure onto the lower back area. D-The massage should be in the lower back where the pain is located.

Cardiovascular changes that cause the foramen ovale to close at birth are a direct result of: a. Increased pressure in the right atrium. b. Increased pressure in the left atrium. c. Decreased blood flow to the left ventricle. d. Changes in the hepatic blood flow.

ANS: b. increased pressure in the left atrium feedback: Since there is an increase blood flow from the lungs to the left atrium, there is an increased pressure, which will close the foramen ovale

A woman's last menstrual period was June 10. The nurse estimates the date of delivery (EDD) to be a. April 7. b. March 17. c. March 27. d. April 17.

ANS: B. March 17 To determine the EDD, the nurse uses the first day of the last menstrual period (June 10), subtracts 3 months (March 10), and adds 7 days (March 17). The year is corrected if needed. April 7 would be subtracting 2 months instead of 3 months and then subtracting 3 days instead of adding 7 days. March is the correct month, but instead of adding 7 days, 17 days were added to get March 27. April 17 is subtracting 2 months instead of 3 months.

A woman gave birth to a healthy 7-pound, 13-ounce infant girl. The nurse suggests that the woman place the infant to her breast within 15 minutes after birth. The nurse knows that breastfeeding is effective during the first 30 minutes after birth because this is the: a. Transition period. b. First period of reactivity. c. Organizational stage. d. Second period of reactivity.

ANS: B. first period of reactivity The first period of reactivity is the first phase of transition and lasts up to 30 minutes after birth. The infant is highly alert during this phase. The transition period is the phase between intrauterine and extrauterine existence. There is no such phase as the organizational stage. The second period of reactivity occurs roughly between 4 and 8 hours after birth, after a period of prolonged sleep.

The nurse caring for women in labor understands that childbirth pain is different from other types of pain in that it is a. More responsive to pharmacologic management b. Associated with a physiologic process c. Designed to make one withdraw from the stimulus d. Less intense

ANS: B.Associated with a physiologic process Feedback A Pain management during labor may affect the course and length of labor. B Childbirth pain is part of a normal process, whereas other types of pain usually signify an injury or illness. C The pain with childbirth is a normal process; it is not caused by the type of injury when withdrawal from the stimuli is seen. D Childbirth pain is not less intense than other types of pain.

It is important for the nurse providing care during labor to be aware that pregnant women can usually tolerate the normal blood loss associated with childbirth because they have a. A higher hematocrit b. Increased blood volume c. A lower fibrinogen level d. Increased leukocytes

ANS: B.Increased blood volume Feedback A The hematocrit decreases with pregnancy due to the high fluid volume. B Women have a significant increase in blood volume during pregnancy. After delivery, the additional circulating volume is no longer necessary. C Fibrinogen levels increase with pregnancy. D Leukocyte levels increase during labor, but that is not the reason for the toleration of blood loss.

A man calls the nurse's station and states that his wife, who delivered 2 days ago, is happy one minute and crying the next. The man says, "She was never like this before the baby was born." The nurse's initial response could be to: a.Tell him to ignore the mood swings, as they will go away. b.Reassure him that this behavior is normal. c.Advise him to get immediate psychological help for her. d.Instruct him in the signs, symptoms, and duration of postpartum blues.

ANS: B.Reassure him that this behavior is normal. Feedback: Before providing further instructions, inform family members of the fact that postpartum blues are a normal process. Telling her partner to "ignore the mood swings" does not encourage further communication and may belittle the husband's concerns. Postpartum blues are usually short-lived; no medical intervention is needed. Client teaching is important; however, the new father's anxieties need to be allayed before he will be receptive to teaching.

The parents of a newborn infant state, "We will probably not have our baby immunized because we are concerned about the risk of our child being injured." What is the nurse's best response? a. "It is your decision." b. "Have you talked with your parents about this? They can probably help you think about this decision." c. "The risks of not immunizing your baby are greater than the risks from the immunizations." d. "You are making a mistake."

ANS: C Feedback A It is the parents' decision not to immunize the child; however, the nurse has a responsibility to inform parents about the risks to infants who are not immunized. B Grandparents can be supportive but are not the primary decision makers for the infant. C Although immunizations have been documented to have a negative effect in a small number of cases, an unimmunized infant is at greater risk for development of complications from childhood diseases than from the vaccines. D Telling parents that they are making a mistake is an inappropriate response.

To provide competent newborn care, the nurse understands that respirations are initiated at birth as a result of a. An increase in the PO2 and a decrease in PCO2 b. The continued functioning of the foramen ovale c. Chemical, thermal, sensory, and mechanical factors d. Drying off the infant

ANS: C Feedback: A The PO2 decreases at birth and the PCO2 increases. B The foramen ovale closes at birth. C A variety of these factors are responsible for initiation of respirations. D Tactile stimuli aid in initiating respirations, but are not the main cause.

Alterations in hormonal balance and mechanical stretching are responsible for several changes in the integumentary system during pregnancy. Stretch marks often occur on the abdomen and breasts. These are referred to as a.Chloasma b.Linea nigra c.Striae gravidarum d.Angiomas

ANS: C C. Striae gravidarum or stretch marks appear in 50% to 90% of pregnant women during the second half of pregnancy. They most often occur on the breasts and abdomen. This integumentary alteration is the result of separation within the underlying connective (collagen) tissue.

Sally comes in for her first prenatal examination. This is her first child. She asks you (the nurse), "How does my baby get air inside my uterus?" The correct response is: a."The baby's lungs work in utero to exchange oxygen and carbon dioxide." b."The baby absorbs oxygen from your blood system." c."The placenta provides oxygen to the baby and excretes carbon dioxide into your bloodstream." d."The placenta delivers oxygen-rich blood through the umbilical artery to the baby's abdomen."

ANS: C "The placenta provides oxygen to the baby and excretes carbon dioxide into your bloodstream." The placenta functions by supplying oxygen and excreting carbon dioxide to the maternal bloodstream. The fetal lungs do not function for respiratory gas exchange in utero. The baby does not simply absorb oxygen from a woman's blood system. Blood and gas transport occur through the placenta. The placenta delivers oxygen-rich blood through the umbilical vein and not the artery.

How can a woman avoid exposing her fetus to teratogens? a.She can update her immunizations during the first trimester of her pregnancy. b.She can use saunas and hot tubs during the winter months only. c.She can use only category A drugs during her pregnancy. d.She can use alcoholic beverages only in the first and third trimesters of pregnancy.

ANS: C She can use only category A drugs during her pregnancy. In well-controlled studies, class A drugs have no demonstrated fetal risk. Immunizations for such things as rubella are contraindicated during pregnancy. Use of saunas and hot tubs are not recommended because maternal hyperthermia is an important teratogen. Alcohol is an environmental substance known to be teratogenic.

Which statement by the nurse is most appropriate to a 15-year-old whose friend has mentioned suicide? a. "Tell your friend to come to the clinic immediately." b. "You need to gather details about your friend's suicide plan." c. "Your friend's threat needs to be taken seriously, and immediate help for your friend is important." d. "If your friend mentions suicide a second time, you will want to get your friend some help."

ANS: C. "Your friend's threat needs to be taken seriously, and immediate help for your friend is important." Feedback A Instructing a 15-year-old to tell a friend to come to the clinic immediately provides the teen with limited information and does not address the concern. B It is important to determine whether a person threatening suicide has a plan of action; however, the best information for the 15-year-old to have is that all threats of suicide should be taken seriously and immediate help is important. C Suicide is the third most common cause of death among American adolescents. A suicide threat from an adolescent serves as a dramatic message to others and should be taken seriously. Adolescents at risk should be targeted for supportive guidance and counseling before a crisis occurs. D It is imperative that help is provided immediately for a teenager who is talking about suicide. Waiting until the teen discusses it a second time may be too late.

To provide safe care for the woman, the nurse understands that which condition is a contraindication for an amniotomy? a. Dilation less than 3 cm b. Cephalic presentation c. -2 station d. Right occiput posterior position

ANS: C. -2 station Feedback A The dilation must be enough to determine labor. B The presenting part should be cephalic. Amniotomy is deferred if the presenting part is higher in the pelvis. C A prolapsed cord can occur if the membranes artificially rupture when the presenting part is not engaged. D This indicates a cephalic presentation, which is appropriate for an amniotomy.

Generally, the earliest age at which puberty begins is _____ years in girls, _____ in boys. a. 13; 13 b. 11; 11 c. 10; 12 d. 12; 10

ANS: C. 10; 12 Puberty signals the beginning of the development of secondary sex characteristics. This begins earlier in girls than in boys. Usually a 2-year difference occurs in the age of onset. Girls and boys do not usually begin puberty at the same age. Girls generally begin puberty 2 years earlier than boys.

While assessing the newborn, the nurse should be aware that the average expected apical pulse range of a full-term, quiet, alert newborn is: a.80 to 100 beats/min. b.100 to 120 beats/min. c.120 to 160 beats/min. d.150 to 180 beats/min.

ANS: C. 120 to 160 beats/min. The average infant heart rate while awake is 120 to 160 beats/min. The newborn's heart rate may be about 85 to 100 beats/min while sleeping. The infant's heart rate typically is a bit higher when alert but quiet. A heart rate of 150 to 180 beats/min is typical when the infant cries.

The nurse administers vitamin K to the newborn for what reason? A. Most mothers have a diet deficient in vitamin K, which results in the infant being deficient. B. Vitamin K prevents the synthesis of prothrombin in the liver and must be given by injection. C. Bacteria that synthesize vitamin K are not present in the newborn's intestinal tract. D. The supply of vitamin K is inadequate for at least 3 to 4 months, and the newborn must be supplemented.

ANS: C. Bacteria that synthesize vitamin K are not present in the newborn's intestinal tract.

Which factor ensures that the smallest anterior-posterior diameter of the fetal head enters the pelvis? a. Descent b. Engagement c. Flexion d. Station

ANS: C. FLEXION Feedback A Descent is the moving of the fetus through the birth canal. B Engagement occurs when the largest diameter of the fetal presenting part has passed the pelvic inlet. C Flexion of the fetal head allows the smallest head diameters pass through the pelvis. D The station is the relationship of the fetal presenting part to the level of the ischial spines.

To increase the absorption of iron in a pregnant woman, an iron preparation should be given with: a. Milk. b. Tea. c. Orange juice. d. Coffee.

ANS: C. Orange juice A vitamin C source may increase the absorption of iron. The calcium and phosphorus in milk decrease iron absorption. Tannin in the tea reduces the absorption of iron. Decreased intake of caffeine is recommended during pregnancy.

As part of Standard Precautions, nurses wear gloves when handling the newborn. The chief reason is: A. To protect the baby from infection. B. It is part of the Apgar protocol. C. To protect the nurse from contamination by the newborn. D. Because the nurse has primary responsibility for the baby during the first 2 hours.

ANS: C. To protect the nurse from contamination by the newborn. Feedback: A. Incorrect: Gloves are worn to protect the nurse from infection until the blood and amniotic fluid are cleaned off the newborn. B. Incorrect: Gloves are worn to protect the nurse from infection until the blood and amniotic fluid are cleaned off the newborn. C. Correct: Gloves are worn to protect the nurse from infection until the blood and amniotic fluid are cleaned off the newborn. D. Incorrect: Gloves are worn to protect the nurse from infection until the blood and amniotic fluid are cleaned off the newborn.

A woman is currently pregnant; she has a 5-year-old son and a 3-year-old daughter born at full term. She had one other pregnancy that terminated at 8 weeks. Her gravida and para are a. gravida 3 para 2. b. gravida 4 para 3. c. gravida 4 para 2. d. gravida 3 para 3.

ANS: C. gravida 4 para 2. She has had four pregnancies, including the current one (gravida 4). She had two pregnancies that terminated after 20 weeks (para 2). The pregnancy that terminated at 8 weeks is classified as an abortion, which is not included in the gravida-para classification.

Firm contractions that occur every 3 minutes and last 100 seconds (I minute 40 seconds) may educe fetal oxygen supply because they: A) Cause fetal bradycardia and reduce oxygen concentration B)Activate the normal variability of the fetal heart C) limit time for oxygen exchange in the placenta D) Suppress the normal variability of the fetal heart.

ANS: C. limit time for oxygen exchange in the placenta

A new mother recalls from prenatal class that she should try to feed her newborn daughter when she exhibits feeding readiness cues rather than waiting until her infant is crying frantically. On the basis of this information, this woman should feed her infant about every 2.5 to 3 hours when she: a.Waves her arms in the air. b.Has hiccups. c.Makes sucking motions. d.Stretches her legs out straight.

ANS: C. makes sucking motions Sucking motions, rooting, mouthing, and hand-to-mouth motions are examples of feeding-readiness cues. Waving the arms in the air, hiccupping, and stretching the legs out straight are not typical feeding-readiness cues.

Nursing care in the fourth trimester includes an important intervention sometimes referred to as taking the time to mother the mother. Specifically this expression refers to: a. Formally initializing individualized care by confirming the woman's and infants identification (ID) numbers on their respective wrist bands. (This is your baby.) b. Teaching the mother to check the identity of any person who comes to remove the baby from the room. (Its a dangerous world out there.) c. Including other family members in the teaching of self-care and child care. (Were all in this together.) d. Nurturing the woman by providing encouragement and support as she takes on the many tasks of motherhood.

ANS: D Many professionals believe that the nurses nurturing and support function is more important than providing physical care and teaching. Matching ID wrist bands is more of a formality, but it is also a get-acquainted procedure. Mothering the mother is more a process of encouraging and supporting the woman in her new role. Having the mother check IDs is a security measure for protecting the baby from abduction. Teaching the wholefamily is just good nursing practice.

A primiparous woman is delighted with her newborn son and wants to begin breastfeeding as soon as possible. The nurse can facilitate the infant's correct latch-on by helping the woman hold the infant: a. With his arms folded together over his chest. b. Curled up in a fetal position. c. With his head cupped in her hand. d. With his head and body in alignment.

ANS: D - With his head and body in alignment. The infant's head and body should be in correct alignment with the mother and the breast during latch-on and feeding. Holding the infant with his arms folded together over his chest, curled up in a fetal position, or with his head cupped in her hand are not ideal positions to facilitate latch-on.

Between 6 and 10 days after conception, the trophoblast secretes enzymes that enable it to burrow into the endometrium until the entire blastocyst is covered. This is termed implantation. Tiny projections then develop out of the trophoblast and extend into the endometrium. These projections are referred to as a. Decidua basalis b. Decidua capsularis c. Decidua vera d. Chorionic villi

ANS: D. Chorionic villi These villi are vascular processes that obtain oxygen and nutrients from the maternal bloodstream and dispose of carbon dioxide and waste products into the maternal blood.

Which term best describes the emotional attitude that one's own ethnic group is superior to others? a. Culture b. Ethnicity c. Superiority d. Ethnocentrism

ANS: D. Ethnocentrism Ethnocentrism is the belief that one's way of living and behaving is the best way. This includes the emotional attitude that the values, beliefs, and perceptions of one's ethnic group are superior to those of others. Culture is a pattern of assumptions, beliefs, and practices that unconsciously frames or guides the outlook and decisions of a group of people. A culture is composed of individuals who share a set of values, beliefs, and practices that serve as a frame of reference for individual perception and judgments. Ethnicity is an affiliation of a set of persons who share a unique cultural, social, and linguistic heritage. Superiority is the state or quality of being superior; it does not include ethnicity.

The most basic information a maternity nurse should have concerning conception is: a.Ova are considered fertile 48 to 72 hours after ovulation b.Sperm remain viable in the woman's reproductive system for an average of 12 to 24 hours c.Conception is achieved when a sperm successfully penetrates the membrane surrounding the ovum d.Implantation in the endometrium occurs 6 to 10 days after conception

ANS: D. Implantation in the endometrium occurs 6 to 10 days after conception After implantation, the endometrium is called the decidua. Ova are considered fertile for about 24 hours after ovulation. Sperm remain viable in the woman's reproductive system for an average of 2 to 3 days. Penetration of the ovum by the sperm is called fertilization. Conception occurs when the zygote, the first cell of the new individual, is formed.

A nurse is completing an assessment. Which of the following data indicate the newborn is adapting to extrauterine life? (Select all that apply.) A. Expiratory grunting B. Inspiratory nasal flaring C. Apnea for 10-second periods D. Obligatory nose breathing E. Crackles and wheezing

ANS: D. Obligatory nose breathing, C. Apnea for 10-second periods Feedback: C. CORRECT: Periods of apnea lasting less than 15 seconds are an expected finding. D. CORRECT: Newborns are obligatory nose breathers A. INCORRECT: Expiratory grunting is a manifestation of respiratory distress. B. INCORRECT: Nasal flaring is a manifestation of respiratory distress. E. INCORRECT: Crackles and wheezing are symptoms of fluid or infection in the lungs.

When explaining twin conception, the nurse points out that dizygotic twins develop from a. A single fertilized ovum and are always of the same sex b. A single fertilized ovum and may be the same sex or different sexes c. Two fertilized ova and are the same sex d. Two fertilized ova and may be the same sex or different sexes

ANS: D. Two fertilized ova and may be the same sex or different sexes feedback: Dizygotic twins are two different zygotes, each conceived from a single ovum and a single sperm. They may

he major source of nutrients in the diet of a pregnant woman should be composed of: a. Simple sugars. b. Fats. c. Fiber. d. Complex carbohydrates.

ANS: D. complex carbohydrates Complex carbohydrates supply the pregnant woman with vitamins, minerals, and fiber. The most common simple carbohydrate is table sugar, which is a source of energy but does not provide any nutrients. Fats provide 9 calories in each gram, in contrast to carbohydrates and proteins, which provide only 4 calories in each gram. Fiber is supplied mainly by the complex carbohydrates.

A newborn is placed under a radiant heat warmer, and the nurse evaluates the infant's body temperature every hour. Maintaining the newborn's body temperature is important for preventing: a.Respiratory depression. b.Cold stress. c.Tachycardia. d.Vasoconstriction.

ANS: b.Cold stress. feedback: Loss of heat must be controlled to protect the infant from the metabolic and physiologic effects of cold stress, and that is the primary reason for placing a newborn under a radiant heat warmer. Cold stress results in an increased respiratory rate and vasoconstriction.

A pregnant woman has come to the emergency department with complaints of nasal congestion and epistaxis. What is the correct interpretation of these symptoms by the practitioner? a. These conditions are abnormal. Refer the client to an ear, nose, and throat specialist. b. Nasal stuffiness and nosebleeds are caused by a decrease in progesterone. c. Estrogen relaxes the smooth muscles in the respiratory tract, so congestion and epistaxis are within normal limits. d. Estrogen causes increased blood supply to the mucous membranes and can result in congestion and nosebleeds.

ANS: D.Estrogen causes increased blood supply to the mucous membranes and can result in congestion and nosebleeds. As capillaries become engorged, the upper respiratory tract is affected by the subsequent edema and hyperemia, which causes these conditions, seen commonly during pregnancy. The client should be reassured that these symptoms are within normal limits. No referral is needed at this time. Progesterone is responsible for the heightened awareness of the need to breathe in pregnancy. Progesterone levels increase during pregnancy. Relaxation of the smooth muscles in the respiratory tract is affected by progesterone.

Which pregnant woman should restrict her weight gain during pregnancy? a. Woman pregnant with twins b. Woman in early adolescence c. Woman shorter than 62 inches or 157 cm d. Woman who was 20 pounds overweight before pregnancy

ANS: D.Woman who was 20 pounds overweight before pregnancy Feedback A A higher weight gain in twin gestations may help prevent low birth weights. B Adolescents need to gain weight toward the higher acceptable range, which will provide for their own growth as well as for fetal growth. C In the past women of short stature were advised to restrict their weight gain; however, evidence to support these guidelines has not been found. D A weight gain of 5 to 9 kg will provide sufficient nutrients for the fetus. Overweight and obese women should be advised to lose weight prior to conception in order to achieve the best pregnancy outcomes.

A new mother states that her infant must be cold because the baby's hands and feet are blue. The nurse explains that this is a common and temporary condition called: a. Acrocyanosis. b. Erythema neonatorum. c. Harlequin color. d. Vernix caseosa.

ANS: a. Acrocyanosis.

A patient who underwent a vaginal delivery 3 hours earlier reports having severe perineal pain. Which would be the first step taken by the nurse in this situation? a.Apply ice packs in the perineum. b.Administer fluids to the patient. c.Administer blood to the patient. d.Refer the patient for hematologic tests.

ANS: a. Apply ice packs in the perineum Feedback: If the patient reports severe perineal pain after vaginal delivery, the nurse should apply ice packs in the first 24 hours to reduce edema, pain, and vulvar irritation. Administering fluids and blood compensates for blood loss in the patient, but they do not reduce pain. Postpartum hematologic studies are performed to assess the consequences of blood loss. This intervention does not reduce pain in the patient.

A new mother asks whether she should feed her newborn colostrum because it is not "real milk." The nurse's best answer includes what information? a. Colostrum is high in antibodies, protein, vitamins, and minerals. b. Colostrum is lower in calories than milk and should be supplemented by formula. c. Giving colostrum is important in helping the mother learn how to breast-feed before she goes home. d. Colostrum is unnecessary for newborns.

ANS: a. Colostrum is high in antibodies, protein, vitamins, and minerals.

If the fundus is palpated on the right side of the abdomen above the expected level, the nurse should suspect that the client has which? a. Distended bladder b. Normal involution c. Been lying on her right side too long d. Stretched ligaments that are unable to support the uterus

ANS: a. Distended bladder The presence of a full bladder will displace the uterus. A palpated fundus on the right side of the abdomen above the expected level is not an expected finding. Position of the client should not alter uterine position. The problem is a full bladder displacing the uterus.

A pregnant woman experiencing nausea and vomiting should: a. Eat dry crackers or toast before arising in the morning. b.Drink coffee or orange juice immediately on arising in the morning. c. Eat only three meals a day so the stomach is empty between meals. d. Drink plenty of fluids with each meal.

ANS: a. Eat dry crackers or toast before arising in the morning.

According to the recommendations of the American Academy of Pediatrics on infant nutrition: a. Infants should be given only human milk for the first 6 months of life. b. Infants fed on formula should be started on solid food sooner than breastfed infants. c. If infants are weaned from breast milk before 12 months, they should receive cow's milk, not formula. d. After 6 months mothers should shift from breast milk to cow's milk.

ANS: a. Infants should be given only human milk for the first 6 months of life.

During which phase of maternal adjustment will the mother relinquish the baby of her fantasies and accept the real baby? a. Letting-go b. Taking-in c. Taking-on d. Taking-hold

ANS: a. Letting-go Accepting the real infant and relinquishing the fantasy infant occurs during the letting-go phase of maternal adjustment. In the taking-in phase, the mother is primarily focused on her own needs. There is no taking-on phase of maternal adjustment. During the taking-hold phase, the mother assumes responsibility for her own care and shifts her attention to the infant.

One of the assessments performed in the delivery room is checking the umbilical cord for blood vessels. Which finding is considered to be within normal limits? a.Two arteries and one vein b.Two arteries and two veins c.Two veins and one artery d.One artery and one vein

ANS: a. Two arteries and one vein feedback: The umbilical cord contains two arteries and one vein to transport blood between the fetus and the placenta. This option is abnormal and may indicate an anomaly. Any option other than two arteries and one vein is considered abnormal and requires further assessment. Two arteries instead of one is considered normal; this infant would require further assessment for other anomalies.

When teaching parents about mandatory newborn screening, it is important for the nurse to explain that the main purpose is to a. Keep the state records updated. b. Allow accurate statistical information. c. Document the number of births. d. Recognize and treat newborn disorders early.

ANS: d. Recognize and treat newborn disorders early.

which statement states the best reason for recommending formula over breastfeeding is that: a.The mother has a medical condition or is taking drugs that could be passed along to the infant via breast milk. b.The mother lacks confidence in her ability to breastfeed. c.Other family members or care providers also need to feed the baby. d.The mother sees bottle-feeding as more convenient.

ANS: a.The mother has a medical condition or is taking drugs that could be passed along to the infant via breast milk. feedback: Breastfeeding is contraindicated when mothers have certain viruses, are undergoing chemotherapy, or are using/abusing illicit drugs. A lack of confidence, the need for others to feed the baby, and the convenience of bottle-feeding are all honest reasons for not breastfeeding, although further education concerning the ease of breastfeeding and its convenience, benefits, and adaptability (expressing milk into bottles) could change some minds. In any case the nurse must provide information in a nonjudgmental manner and respect the mother's decision. Nonetheless, breastfeeding is definitely contraindicated when the mother has medical or drug issues of her own.

The breast-feeding mother should be taught a safe method to remove her breast from the baby's mouth. Which suggestion by the nurse is most appropriate? a. Slowly remove the breast from the baby's mouth when the infant has fallen asleep and the jaws are relaxed. b. Break the suction by inserting your finger into the corner of the infant's mouth. c. A popping sound occurs when the breast is correctly removed from the infant's mouth. d. Elicit the Moro reflex in the baby to wake the baby up, and remove the breast when the baby cries.

ANS: b. Break the suction by inserting your finger into the corner of the infant's mouth.

The process in which bilirubin is changed from a fat-soluble product to a water-soluble product is known as: a. Enterohepatic circuit. b. Conjugation of bilirubin. c. Deconjugation of bilirubin. d. Albumin binding.

ANS: b. Conjugation of bilirubin.

When the nurse interviews an adolescent, which of the following is especially important? a.Focus the discussion on the peer group. b.Display a genuine interest in the adolescent. c.Emphasize that confidentiality will always be maintained. d.Use the same type of language as the adolescent.

ANS: b. Display a genuine interest in the adolescent.

14. When caring for a newly delivered woman, the nurse is aware that the best measure to prevent abdominal distention after a cesarean birth is a. Rectal suppositories b. Early and frequent ambulation c. Tightening and relaxing abdominal muscles d. Carbonated beverages

ANS: b. Early and frequent ambulation Feedback: Activity can aid the movement of accumulated gas in the gastrointestinal tract.

When teaching parents about their newborn's transition to extrauterine life, the nurse explains which organs are nonfunctional during fetal life. They are the a. Kidneys and adrenals b. Lungs and liver c. Eyes and ears d. Gastrointestinal system

ANS: b. Lungs and liver Feedback: A Kidneys and adrenals function during fetal life. The fetus continuously swallows amniotic fluid, which is filtered through the kidneys. B Most of the fetal blood flow bypasses the nonfunctional lungs and liver. C Near term, the eyes are open and the fetus can hear. D The gastrointestinal system functions during fetal life

The postpartum woman continually repeats the story of her labor, delivery, and recovery experience. What is this new mother attempting to achieve with this behavior? a. Providing others with her knowledge of events b. Making the birth experience real c. Taking hold of the events leading up to her labor and delivery d. Accepting her response to labor and delivery

ANS: b. Making the birth experience real

As relates to rubella and Rh issues, nurses should be aware that: a. Breastfeeding mothers cannot be vaccinated with the live attenuated rubella virus. b. Women should be warned that the rubella vaccination is teratogenic, and that they must avoid pregnancy for 1 month after vaccination. c. Rh immune globulin is safely administered intravenously because it cannot harm a nursing infant. d. Rh immune globulin boosts the immune system and thereby enhances the effectiveness of vaccinations.

ANS: b. Women should be warned that the rubella vaccination is teratogenic, and that they must avoid pregnancy for 1 month after vaccination.

a nurse is providing care to four clients on the postpartum unit. Which of the following clients is at greatest risk for developing a postpartum infection? a. a client who has an episiotomy that is erythematous and has extended into a third‑degree laceration b. a client who does not wash her hands between perineal care and breastfeeding c. a client who is not breastfeeding and is using measures to suppress lactation d. a client who has a cesarean incision that is well‑approximated with no drainage

ANS: b. a client who does not wash her hands between perineal care and breastfeeding feedback: a. an episiotomy with a laceration is at risk for an infection, but there is a client who is at greater risk for a postpartum infection. B. CORRECT: the client who does not wash her hands between perineal care and breastfeeding is at an increased risk for developing mastitis. therefore, she is most at risk for developing a postpartum infection. C. a client who is suppressing lactation (increases the risk of milk stasis) is at risk for an infection, but there is a client who is at greater risk for a postpartum infection. d. a client who has an abdominal incision is at risk for an infection, but there is a client who is at greater risk for a postpartum infection."

A woman who is gravida 3, para 2, enters the intrapartum unit. The most important nursing assessments are: a. contraction pattern, amount of discomfort, and pregnancy history. b. fetal heart rate, maternal vital signs, and the woman's nearness to birth. c. last food intake, when labor began, and cultural practices the couple desires. d. identification of ruptured membranes, the woman's gravida and para, and her support person.

ANS: b. fetal heart rate, maternal vital signs, and the woman's nearness to birth All options describe relevant intrapartum nursing assessments, but the focus assessment has priority. If the maternal and fetal conditions are normal and birth is not imminent, other assessments can be performed in an unhurried manner. Contraction pattern, amount of discomfort, and pregnancy history are important nursing assessments but do not take priority if the birth is imminent. Last food intake, when labor began, and cultural practices the couple desires is an assessment that can occur later in the admission process, if time permits. Identification of ruptured membranes, the woman's gravida and para, and her support person are assessments that can occur later in the admission process if time permits.

The nurse examines a woman 1 hour after birth. The woman's fundus is boggy, midline, and 1 cm below the umbilicus. Her lochial flow is profuse, with two plum-sized clots. The nurse's initial action would be to: a.place her on a bedpan to empty her bladder. b.massage her fundus. c.call the physician. d.administer Methergine, 0.2 mg IM, which has been ordered prn.

ANS: b.massage her fundus. Feedback: There is no indication of a distended bladder; thus having the woman urinate will not alleviate the problem. A boggy or soft fundus indicates that uterine atony is present. This is confirmed by the profuse lochia and passage of clots. The first action would be to massage the fundus until firm. The physician can be called after massaging the fundus, especially if the fundus does not become or remain firm with massage. Methergine can be administered after massaging the fundus, especially if the fundus does not become or remain firm with massage.

The school nurse tells adolescents in the clinic that confidentiality and privacy will be maintained unless a life-threatening situation arises. This practice is: a. Not appropriate in a school setting. b. Never appropriate because adolescents are minors. c. Important in establishing trusting relationships. d. Suggestive that the nurse is meeting his or her own needs.

ANS: c - Important in establishing trusting relationships.

A nurse is preparing to perform a fundal assessment on a postpartum client. The initial nursing action in performing this assessment is which of the following? a.Ask the client to turn on her side b. Ask the client to lie flat on her back with the knees and legs flat and straight. c.Ask the mother to urinate and empty her bladder d.Massage the fundus gently before determining the level of the fundus.

ANS: c. Ask the mother to urinate and empty her bladder Before starting the fundal assessment, the nurse should ask the mother to empty her bladder so that an accurate assessment can be done. When the nurse is performing fundal assessment, the nurse asks the woman to lie flat on her back with the knees flexed. Massaging the fundus is not appropriate unless the fundus is boggy and soft, and then it should be massaged gently until firm.

The nurse hears a primiparous woman talking to her son and telling him that his chin is just like his dad's chin. This woman's statement reflects: a. Mutuality. b. Synchrony. c. Claiming. d. Reciprocity.

ANS: c. Claiming

Postpartal overdistention of the bladder and urinary retention can lead to which complication? a. Postpartum hemorrhage and eclampsia b. Fever and increased blood pressure c. Postpartum hemorrhage and urinary tract infection d. Urinary tract infection and uterine rupture

ANS: c. Postpartum hemorrhage and urinary tract infection

The normal term infant has little difficulty clearing its airway after birth. Most secretions are brought up to the oropharynx by the cough reflex. However, if the infant has excess secretions, the mouth and nasal passages can easily be cleared with a bulb syringe. When instructing parents on the correct use of this piece of equipment, it is important that the nurse teach them to a. Avoid suctioning the nares. b. Insert the compressed bulb into the center of the mouth. c. Suction the mouth first. d. Remove the bulb syringe from the crib when finished.

ANS: c. Suction the mouth first.

When providing care to the prenatal patient, the nurse understands that pica is defined as: a.Intolerance of milk products c.Ingestion of nonfood substances b.Iron deficiency anemia d.Episodes of anorexia and vomiting

ANS: c.Ingestion of nonfood substances The practice of eating substances not normally thought of as food is called pica. Clay or dirt and solid laundry starch are the substances most commonly ingested. Intolerance of milk products is referred to as lactose intolerance. Pica may produce iron deficiency anemia if proper nutrition is decreased. Pica is not related to anorexia and vomiting.

A newborn is jaundiced and receiving phototherapy via ultraviolet bank lights. An appropriate nursing intervention when caring for an infant with hyperbilirubinemia and receiving phototherapy by this method would be to: a.Apply an oil-based lotion to the newborn's skin to prevent dying and cracking. b.Limit the newborn's intake of milk to prevent nausea, vomiting, and diarrhea. c.Place eye shields over the newborn's closed eyes. d.Change the newborn's position every 4 hours.

ANS: c.Place eye shields over the newborn's closed eyes. The infant's eyes must be protected by an opaque mask to prevent overexposure to the light. Eye shields should cover the eyes completely but not occlude the nares. Lotions and ointments should not be applied to the infant because they absorb heat, and this can cause burns. The lights increase insensible water loss, placing the infant at risk for fluid loss and dehydration. Therefore, it is important that the infant be adequately hydrated. The infant should be turned every 2 hours to expose all body surfaces to the light.

While evaluating the reflexes of a newborn, the nurse notes that with a loud noise the newborn symmetrically abducts and extends his arms, his fingers fan out and form a "C" with the thumb and forefinger, and he has a slight tremor. The nurse would document this finding as a positive: a. Tonic neck reflex. b. Glabellar (Myerson) reflex. c. Babinski reflex. d. Moro reflex.

ANS: d. MORO REFLEX

Parents have been asked by the neonatologist to provide breast milk for their newborn son, who was born prematurely at 32 weeks of gestation. The nurse who instructs them regarding pumping, storing, and transporting the milk needs to assess their knowledge of lactation. Which statement is valid? a. The mother should only pump as much milk as the infant can drink. b. A glass of wine just before pumping will help reduce stress and anxiety. c. The mother should pump every 2 to 3 hours, including during the night. d. Premature infants more easily digest breast milk than formula.

ANS: d. Premature infants more easily digest breast milk than formula.

The nurse assesses a postpartum patient several hours after delivery and suspects that the uterus is subinvoluted. What could be a potential etiology for this finding? a. Estrogen levels b. Progesterone levels c. Impaired platelet aggregation d. Retained placental fragments

ANS: d. RETAINED PLACENTAL FRAGMENTS Retained placental fragments or infection cause subinvolution of the uterus. Therefore the nurse should assess the patient for any placental fragments in the uterus. Estrogen and progesterone stimulate massive growth of the uterus during pregnancy. In the postpartum stage, the hormone levels are reduced and, therefore, do not affect involution of the uterus. Platelet aggregation causes uterine muscle contraction, but it does not result in involution of the uterus.

A nursing student is helping the nursery nurses with morning vital signs. A baby born 10 hours ago via cesarean section is found to have moist lung sounds. What is the best interpretation of these data? a. The nurse should notify the pediatrician stat for this emergency situation. b. The neonate must have aspirated surfactant. c. If this baby was born vaginally, it could indicate a pneumothorax. d. The lungs of a baby delivered by cesarean section may sound moist for 24 hours after birth.

ANS: d. The lungs of a baby delivered by cesarean section may sound moist for 24 hours after birth. feedback: A This is a common condition for infants delivered by cesarean section. B Surfactant is produced by the lungs, so aspiration is not a concern. C It is common to have some fluid left in the lungs; this will be absorbed within a few hours. D The condition will resolve itself within a few hours. For this common condition of newborns, surfactant acts to keep the expanded alveoli partially open between respirations. In vaginal births, absorption of remaining lung fluid is accelerated by the process of labor and delivery. Remaining lung fluid will move into interstitial spaces and be absorbed by the circulatory and lymphatic systems.

While assessing her patient, what does the nurse interpret as a positive sign of pregnancy? a. Fetal movement felt by the woman b. Amenorrhea c. Breast changes d. Visualization of fetus by ultrasound

ANS: d. Visualization of fetus by ultrasound The only positive signs of pregnancy are auscultation of fetal heart tones, visualization of the fetus by ultrasound, and fetal movement felt by the examiner. Fetal movement felt by the woman, amenorrhea, and breast changes are all presumptive signs.

Which vitamins or minerals may lead to congenital malformations of the fetus if taken in excess by the mother? a.Zinc b.Vitamin D c.Folic acid d.Vitamin A

ANS: d. Vitamin A If taken in excess, vitamin A causes a number of problems. An analog of vitamin A appears in prescribed acne medications, which must not be taken during pregnancy. Zinc, vitamin D, and folic acid are all vital to good maternity and fetal health and are highly unlikely to be consumed in excess.

What intervention does the nurse perform to suppress lactation in a patient who had a stillbirth? a.Run warm water over the patient's breasts. b.Administer strong analgesics. c.Administer oral and intravenous fluids. d.Advise the patient to wear a breast binder for the first 72 hours after giving birth.

ANS: d.Advise the patient to wear a breast binder for the first 72 hours after giving birth. feedback: Suppression of lactation is recommended in cases of neonatal death. To suppress lactation, the nurse should advise the patient to wear a breast binder continuously for the first 72 hours after delivery. Running warm water over the breast stimulates lactation. Mild analgesics can be administered to reduce breast engorgement, but they are not used to suppress lactation. Administration of oral or intravenous fluids may stimulate lactation.

The nurse's initial action when caring for an infant with a slightly decreased temperature is to A. Notify the physician immediately B. Wrap the infant in two warmed blankets and place a cap on the head C. Tell the mother that the infant must be kept in the nursery and observed for the next 4 hours D. Change the formula, because this is a sign of formula intolerance

Answer: B Rationale: A. A slight decrease in temperature can be handled by nursing intervention. B. Warmed blankets will help to increase the infant's temperature by conduction, and the cap will prevent further heat loss from the head. C. With appropriate nursing interventions, the baby may be taken to the mother with instructions. D. There is no relationship between formula intolerance and decreased temperature.

Which behavior by parents or teachers will best assist the child in negotiating the developmental task of industry? a. Identifying failures immediately and asking the child's peers for feedback b. Structuring the environment so that the child can master tasks c. Completing homework for children who are having difficulty in completing assignments d. Decreasing expectations to eliminate potential failures

B Structuring the environment so that the child can master tasks Feedback A Asking peers for feedback reinforces the child's feelings of failure. B The task of the caring teacher or parent is to identify areas in which a child is competent and to build on successful experiences to foster feelings of mastery and success. Structuring the environment to enhance self-confidence and to provide the opportunity to solve increasingly more complex problems will promote a sense of mastery. C When teachers or parents complete children's homework for them, it sends the message that you do not trust them to do a good job. Providing assistance and suggestions and praising their best efforts are more appropriate. D Decreasing expectations to eliminate failures will not promote a sense of achievement or mastery.

Which of these conditions may cause the fetal heart rate to be lower during labor? (Choose all that apply.) A. Stimulation of the sympathetic nervous system B. Stimulation of the baroreceptors, which in turn stimulates the vagus nerve C. Prolonged hypoxia, hypercapnia, and acidosis D. Stimulation of the parasympathetic nervous system

B,C,D Stimulation of the baroreceptors and the parasympathetic nervous system will lower the heart rate. Initial decreased oxygen content and increased carbon dioxide content will trigger an increase in the heart rate. However, if this condition continues, the heart rate will lower.

The mother of a school-age child tells the school nurse that she and her spouse are going through a divorce. The child has not been doing well in school and sometimes has trouble sleeping. The nurse should recognize this as: a. Indicative of maladjustment. b. Common reaction to divorce. c. Suggestive of lack of adequate parenting. d. Unusual response that indicates need for referral.

B. Common reaction to divorce. Parental divorce affects school-age children in many ways. In addition to difficulties in school, they often have profound sadness, depression, fear, insecurity, frequent crying, loss of appetite, and sleep disorders. Uncommon responses to parental divorce include indications of maladjustment, the suggestion of lack of adequate parenting, and the need for referral.

The nurse is teaching parents of a toddler about language development. Which statement best identifies the characteristics of language development in a toddler? a. Language development skills slow during the toddler period. b. The toddler understands more than he can express. c. Most of the toddler's speech is not easily understood. d. The toddler's vocabulary contains approximately 600 words.

B. The toddler understands more than he can express. Feedback A Although language development varies in relationship to physical activity, language skills are rapidly accelerating by 15 to 24 months of age. B The toddler's ability to understand language (receptive language) exceeds the child's ability to speak it (expressive language). C By 2 years of age, 60% to 70% of the toddler's speech is understandable. D The toddler's vocabulary contains approximately 300 or more words.

When teaching injury prevention during the school-age years, the nurse should include: a. Teaching the need to fear strangers. b. Teaching basic rules of water safety. c. Avoiding letting children cook in microwave ovens. d. Cautioning children against engaging in competitive sports.

B. teaching basic rules of water safety (Water safety instruction is an important source of injury prevention at this age. The child should be taught to swim, select safe and supervised places to swim, swim with a companion, check sufficient water depth for diving, and use an approved flotation device. Teach stranger safety, not fearing strangers. This includes not going with strangers, not having personalized clothing in public places, having children tell parents if anyone makes them uncomfortable, and teaching children to say "no" in uncomfortable situations. Teach children safe cooking methods. Caution against engaging in hazardous sports, such as those involving trampolines.)

Which expected outcome is developmentally appropriate for a hospitalized 4-year-old child? a. The child will be dressed and fed by the parents. b. The child will independently ask for play materials or other personal needs. c. The child will be able to verbalize an understanding of the reason for the hospitalization. d. The child will have a parent stay in the room at all times.

B. the child will independently ask for play materials or other personal needs

During the preschool period, the emphasis of injury prevention should be placed on: a. Constant vigilance and protection. b. Punishment for unsafe behaviors. c. Education for safety and potential hazards. d. Limitation of physical activities.

C Education for safety and potential hazards. Education for safety and potential hazards is appropriate for preschoolers because they can begin to understand dangers. Constant vigilance and protection is not practical at this age since preschoolers are becoming more independent. Punishment may make children scared of trying new things. Limitation of physical activities is not appropriate.

Which action is most likely to encourage parents to talk about their feelings related to their child's illness? a. Be sympathetic. b. Use direct questions. c. Use open-ended questions. d. Avoid periods of silence.

C Use open-ended questions. Closed-ended questions should be avoided when attempting to elicit parents' feelings. Open-ended questions require the parent to respond with more than a brief answer. Sympathy is having feelings or emotions in common with another person rather than understanding those feelings (empathy). Sympathy is not therapeutic in the helping relationship. Direct questions may obtain limited information. In addition, the parent may consider them threatening. Silence can be an effective interviewing tool. It allows sharing of feelings in which two or more people absorb the emotion in depth. Silence permits the interviewee to sort out thoughts and feelings and search for responses to questions.

What does the nurse need to know when observing a chronically ill child at play? a. Play is not important to hospitalized children. b. Children need to have structured play periods. c. Children's play is a form of communication. d. Play is to be discouraged because it tires hospitalized children. C. children's play is a form of communication

C. children's play is a form of communication

The nurse is planning a teaching session for a young child and her parents. According to Piaget's theory, the period of cognitive development in which the child is able to distinguish between concepts related to fact and fantasy, such as human beings are incapable of flying like birds, is the _______ period of cognitive development. a. Sensorimotor b. Formal operations c. Concrete operations d. Preoperational

C. concrete operations

The role of the peer group in the life of school-age children is that it: a. Gives them an opportunity to learn dominance and hostility. b. Allows them to remain dependent on their parents for a longer time. c. Decreases their need to learn appropriate sex roles. d. Provides them with security as they gain independence from their parents.

D - Provides them with security as they gain independence from their parents. Peer-group identification is an important factor in gaining independence from parents. Through peer relationships, children learn ways to deal with dominance and hostility. They also learn how to relate to people in positions of leadership and authority and explore ideas and the physical environment. Peer-group identification helps in gaining independence rather than remaining dependent. A child's concept of appropriate sex roles is influenced by relationships with peers.

When providing labor support, the nurse knows that which fetal position might cause the laboring woman more back discomfort? a. Right occiput anterior b. Left occiput anterior c. Right occiput transverse d. Left occiput posterior

D. Left occiput posterior In the left occiput posterior position, each contraction pushes the fetal head against the mother's sacrum, which results in intense back discomfort.

A patient who had a cesarean birth is immobile in the immediate postoperative period. Which risk is increased in the patient as a result of the hypercoagulable state of the puerperal period? A. Thrombocytosis B. Thrombophlebitis C. Thrombocytopenia D. Thromboembolism

D. THROMBOEMBOLISM Thromboembolism refers to the condition in which a blood vessel is blocked by a blood clot. As the postpartum period is characterized by a hypercoagulation state, the patient is at risk of thromboembolism. Thrombophlebitis is the inflammation of the vein and is not associated with hypercoagulation. Thrombocytopenia refers to the condition in which low levels of platelet are found in the blood. Thrombocytosis is a condition characterized by a significant increase in the number of platelets in the blood.

While teaching an early pregnancy class, the nurse explains that the morula is a a. Fertilized ovum before mitosis begins b. Flattened disc-shaped layer of cells within a fluid-filled sphere c. Double layer of cells that becomes the placenta d. Solid ball composed of the first cells formed after fertilization

D.Solid ball composed of the first cells formed after fertilization feedback: D The morula is so named because it resembles a mulberry. It is a solid ball of 12 to 16 cells that develops after fertilization

Fertilization takes place in the ____________. A. Uterus B. Cervix C. Falloplian tube D. Ovaries

FALLOPIAN TUBE

In order to prevent neural tube defects, updated recommendations include an intake of 0.4mg to 0.8mg of __________ each day from one month prior to conception until 8 to 10 weeks of pregnancy

FOLIC ACID

Very fine hairs called ___________, appear first on the fetus's eyebrows and upper lip at 12 weeks of gestation. By 20 weeks, they cover the entire body. By 28 weeks, the scalp hair is longer than these fine hairs, which thin and may disappear by term gestation.

LANUGO

Which term is used to describe the period between childbirth and the reversal of the mother's uterus into its nonpregnant state? a. Lochia b.Involution c.Puerperium d.Subinvolution

Puerperium The time interval between the birth of the newborn and the return of the patient's reproductive organs to their normal nonpregnant state is termed puerperium, or the fourth trimester of pregnancy. This period usually lasts for 6 weeks. Lochia refers to uterine discharge after birth. Involution refers to the return of the uterus to its nonpregnant state after birth. Subinvolution refers to the failure of the uterus to return to its nonpregnant state after childbirth.

A woman provides the nurse with the following obstetrical history: Delivered a son, now 7 years old, at 28 weeks' gestation; delivered a daughter, now 5 years old, at 39 weeks' gestation; had a miscarriage 3 years ago, and had a first-trimester abortion 2 years ago. She is currently pregnant. Which of the following portrays an accurate picture of this woman's gravidity and parity? 1. 42121. 2. 41212. 3. 51122. 4. 52211.

This accurately reflects this woman's gravidity and parity—G5 P1122.

After you complete your nutritional counseling for a pregnant woman, you ask her to repeat your instructions so you can assess her understanding of the instructions given. Which statement indicates that she understands the role of protein in her pregnancy? a. "Protein will help my baby grow." b. "Eating protein will prevent me from becoming anemic." c. "Eating protein will make my baby have strong teeth after he is born." d. "Eating protein will prevent me from being diabetic."

a. "Protein will help my baby grow."

Which aspect of cognition develops during adolescence? a. Capability to use a future time perspective b. Ability to place things in a sensible and logical order c. Ability to see things from the point of view of another d. Progress from making judgments based on what they see to making judgments based on what they reason

a. Capability to use a future time perspective

What type of sexual abuse is this ? Photographing an any media, sexual acts involving children, and may distribute such material. a. Child pornography b. Incest c. Exhibitionism d. Child prostitution e. Molestation f. Pedophilia

a. Child pornography

While developing an intrapartum care plan for the patient in early labor, it is important that the nurse recognize that psychosocial factors may influence a woman's experience of pain. These include (Select all that apply): a. Culture. b. Anxiety and fear. c. Previous experiences with pain. d. Intervention of caregivers. e. Support systems.

a. Culture. b. Anxiety and fear. c. Previous experiences with pain. e. Support systems.

A primigravida at 39 weeks of gestation is observed for 2 hours in the intrapartum unit. The fetal heart rate has been normal. Contractions are 5 to 9 minutes apart, 20 to 30 seconds in duration, and of mild intensity. Cervical dilation is 1 to 2 cm and uneffaced (unchanged from admission). Membranes are intact. The nurse should expect the woman to be: a. Discharged home to await the onset of true labor b. Admitted for extended observation c. Discharged home with a sedative d. Admitted and prepared for a cesarean birth

a. Discharged home to await the onset of true labor

Which is the most developmentally appropriate intervention when working with the hospitalized adolescent? a. Encourage peers to call and visit when the adolescent's condition allows. b. Encourage the adolescent's friends to continue with their daily activities; the adolescent has concrete thinking and will understand. c. Discourage questions and concerns about the effects of the illness on the adolescent's appearance. d. Ask the parents how the adolescent usually copes in new situations.

a. Encourage peers to call and visit when the adolescent's condition allows. Feedback A The peer group is important to the adolescent's sense of belonging and identity; therefore separation from friends is a major source of anxiety for the hospitalized adolescent. B Adolescents should have advanced beyond concrete thinking. In addition, hospitalized adolescents may be upset if their friends continue with daily activities without them. Communication, interacting, and meeting with friends will be important. C Questions and concerns should be encouraged regarding the adolescent's appearance and the effects of illness on appearance. D How the adolescent copes should be asked directly of the adolescent.

Which woman is most likely to have severe afterbirth pains and request a narcotic analgesic? a. Gravida 5, para 5 b. Woman who is bottle-feeding her first child c. Primipara who delivered a 7-lb boy d. Woman who wishes to breastfeed as soon as her baby is out of the neonatal intensive care unit

a. Gravida 5, para 5

The most common cause of death in the adolescent age-group involves: a. Motor vehicles b. Drownings c. Drug Overdose d. Firearms

a. Motor vehicles

Injuries claim many lives during adolescence. Which factors contribute to early adolescents engaging in risk-taking behaviors (select all that apply)? a. Peer pressure b. A desire to master their environment c. Engagement in the process of separation from their parents d. A belief that they are invulnerable e. Impulsivity

a. Peer pressure d. A belief that they are invulnerable e. Impulsivity

At some hospitals in the US new mothers are given formula gift packs at discharge. Having been given the gift pack by hospital staff leads parents to believe that formula will be necessary even for breastfeeding mothers. Is this statement true or false? a. True. b. False

a. TRUE

When teaching contraception, the nurse must be able to effectively communicate the nuances of contraception. An ovum has the capacity to be fertilized for only 24 hours, whereas a sperm may remain fertile for up to 80 hours. Is this statement true or false? a. True b. False

a. TRUE

In terms of cognitive development, the 5-year-old child would be expected to: a. Use magical thinking. b. Think abstractly. c. Understand conservation of matter. d. Be able to comprehend another person's perspective

a. Use magical thinking. Magical thinking is believing that thoughts can cause events. Abstract thought does not develop until school-age years. The concept of conservation is the cognitive task of school-age children ages 5 to 7 years. Five-year-olds cannot understand another's perspective.

what are the warning signs of sexual abuse seen in the school age child? select all that apply: a. signs of depression or PTSD b. Poor relationship with peers c. Tachycardia and tachypnea d. Unexplained weight gain e. Pain, itching, or burning in the genital area

a. signs of depression or PTSD b. Poor relationship with peers e. Pain, itching, or burning in the genital area

People who have two copies of the same abnormal autosomal dominant gene will usually be: a.More severely affected by the disorder than people with one copy of the gene. b.Infertile and unable to transmit the gene. c.Carriers of the trait but not affected with the disorder. d.Mildly affected with the disorder.

a.More severely affected by the disorder than people with one copy of the gene. People who have two copies of an abnormal gene are usually more severely affected by the disorder because they have no normal gene to maintain normal function. Infertility may or may not be caused by chromosomal defects. A carrier of a trait has one recessive gene. Those mildly affected with the disorder will have only one copy of the abnormal gene.

Health teaching during routine prenatal care includes providing patients with the recommended weight gain during pregnancy. For a woman with a single fetus who begins pregnancy at a normal weight, this amount is ________lb. a. 10 to 15 b. 15 to 20 c. 37 to 50 d. 28 to 40

d. 28 to 40

Which statement made by a mother is consistent with a developmental delay? a. "I have noticed that my 9-month-old infant responds consistently to the sound of his name." b. "I have noticed that my 12-month-old child does not get herself to a sitting position or pull to stand." c. "I am so happy when my 1 1/2-month-old infant smiles at me." d. "My 5-month-old infant is not rolling over in both directions yet."

b. "I have noticed that my 12-month-old child does not get herself to a sitting position or pull to stand." Feedback A An infant who responds to his name at 9 months of age is demonstrating abilities to both hear and interpret sound. B Critical developmental milestones for gross motor development in a 12-month-old include standing briefly without support, getting to a sitting position, and pulling to stand. If a 12-month-old child does not perform these activities, it may be indicative of a developmental delay. C A social smile is present by 2 months of age. D Rolling over in both directions is not a critical milestone for gross motor development until the child reaches 6 months of age.

Based on concepts related to the normal growth and development of children, which child would have the most difficulty with separation from family during hospitalization? a. A 5-month-old infant b. A 15-month-old toddler c. A 4-year-old child d. A 7-year-old child

b. A 15-month-old toddler Feedback A Infants younger than 6 months of age will generally adapt to hospitalization if their basic needs for food, warmth, and comfort are met. B Separation is the major stressor for children hospitalized between ages 6 and 30 months. C Although separation anxiety occurs in hospitalized preschoolers, it is usually less obvious and less serious than that experienced by the toddler. D The school-age child is accustomed to separation from parents. Although hospitalization is a stressor, the 7-year-old child will have less separation anxiety than a 15-month-old toddler.

Physiologic anemia often occurs during pregnancy as a result of: a. Inadequate intake of iron b. Dilution of hemoglobin concentration. c. The fetus establishing iron stores. d. Decrease production of erythrocytes

b. Dilution of hemoglobin concentration.

The nurse who has chosen to work in the field of genetics must have a clear understanding of genetic abnormalities when counseling parents. The nurse is explaining a single gene abnormality to new parents and states, "Although you are second cousins, you are at no greater risk for having a child with a genetic disorder". a. True b. False

b. FALSE

What type of sexual abuse is this? Any physical sexual activity between family members ( blood relationship not required). a. Child pornography b. Incest c. Exhibitionism d. Child prostitution e. Molestation f. Pedophilia

b. Incest

A teen asks a nurse, "What is physical dependence in substance abuse?" Which is the correct response by the nurse? a. Problem that occurs in conjunction with addiction b. Involuntary physiologic response to drug c. Culturally defined use of drugs for purposes other than accepted medical purposes d. Voluntary behavior based on psychosocial needs

b. Involuntary physiologic response to drug

What is descriptive of the play of school-age children? a. Individuality in play is better tolerated than at earlier ages. b. Knowing the rules of a game gives an important sense of belonging. c. They like to invent games, making up the rules as they go. d. Team play helps children learn the universal importance of competition and winning

b. Knowing the rules of a game gives an important sense of belonging.

A 17-year-old tells the nurse that he is not having sex because it would make his parents very angry. This response indicates that the adolescent has a developmental lag in which area? a. Cognitive development b. Moral development c. Psychosocial development d. Psychosexual development

b. Moral development Feedback A Cognitive development is related to moral development, but it is not the pivotal point in determining right and wrong behaviors. B The appropriate moral development for a 17-year-old would include evidence that the teenager has internalized a value system and does not depend on parents to determine right and wrong behaviors. Adolescents who remain concrete thinkers may never advance beyond conformity to please others and avoid punishment. C Identity formation is the psychosocial development task. Energy is focused within the adolescent, who exhibits behavior that is self-absorbed and egocentric. D Although a task during adolescence is the development of a sexual identity, the teenager's dependence on the parents' sanctioning of right or wrong behavior is more appropriately related to moral development.

The psychologic effects of being obese during adolescence include which of the following? a. Sexual promiscuity b. Poor body image c. Feelings of contempt for thin peers d. Accurate body image but self-deprecating attitude

b. Poor body image Common emotional consequences of obesity include poor body image, low self-esteem, social isolation, and feelings of depression and isolation.

The maternity nurse understands that vascular volume increases 40% to 60% during pregnancy to: a. Compensate for decreased renal plasma flow. b. Provide adequate perfusion of the placenta. c. Eliminate metabolic wastes of the mother. d. Prevent maternal and fetal dehydration.

b. Provide adequate perfusion of the placenta.

The nurse observes several interactions between a postpartum woman and her new son. What behavior, if exhibited by this woman, would the nurse identify as a possible maladaptive behavior regarding parent-infant attachment? a. Talks and coos to her son b. Seldom makes eye contact with her son c. Cuddles her son close to her d. Tells visitors how well her son is feeding

b. Seldom makes eye contact with her son

An important nursing intervention is maintaining safe glucose levels in the newborn. A common practice is to feed infants either breast milk or formula if glucose screening shows results of 40 to 45 mg/dL or less. is this statement true or false? a. True b. False

b. TRUE

Why should continuous electronic fetal monitoring usually used when oxytocin is administered? a. The mother may become hypotensive. b. Uteroplacental exchange may be compromised. c. Maternal fluid volume deficit may occur. d. Fetal chemoreceptors are stimulated.

b. Uteroplacental exchange may be compromised.

A nurse is educating a group of 8 year-olds what information should be included? a. you should get 30 minutes of exercise a day. b. eating foods that are colorful and from natural sources are good c. drinking fluids is important, so it is better to drink diet soda to reduce the calories d. you should brush and floss your teeth at lest once a day for overall health

b. eating foods that are colorful and from natural sources are good

A postpartum patient asks, "Will these stretch marks ever go away?" Which is the nurse's best response? a."No, never." b."Yes, eventually." c."They will fade to silvery lines but won't disappear completely." d."They will continue to fade and should be gone by your 6-week checkup."

c. "They will fade to silvery lines but won't disappear completely."

The nurse is performing a comprehensive physical examination on a young child in the hospital. At what age can the nurse expect a child's head and chest circumferences to be almost equal? a. Birth b. 6 months c. 1 year d. 3 years

c. 1 year Feedback A Head circumference is larger than chest circumference until approximately 12 months of age. B Chest circumference is smaller than head circumference until approximately 1 year of age. C Head and chest measurements are almost equal at 1 year of age. D By 3 years of age, the chest circumference exceeds the head circumference.

What type of sexual abuse is this? Indecent exposure of unusually exposed body parts to children. a. Child pornography b. Incest c. Exhibitionism d. Child prostitution e. Molestation f. Pedophilia

c. Exhibitionism

What is the single most important factor to consider when communicating with children? a. The child's physical condition b. Presence or absence of the child's parent c. The child's developmental level d. The child's nonverbal behavior

c. The child's developmental level The nurse must be aware of the child's developmental stage to engage in effective communication. The use of both verbal and nonverbal communication should be appropriate to the developmental level. Although the child's physical condition is a consideration, developmental level is much more important. The parents' presence is important when communicating with young children but may be detrimental when speaking with adolescents. Nonverbal behaviors will vary in importance, based on the child's developmental level

A 6 year old has been learning about healthy eating in school. the school nurse knows it is important for the parents to receive information on healthy eating habits. One area the nurse plans to highlight is the difference between simple and complex carbohydrates. select complex carbohydrate food to include on the handout. select all that apply: a. white rice b. grapes c. starchy vegetables d. brown rice e. whole grain cereal

c. starchy vegetables d. brown rice e. whole grain cereal

For which client should the oxytocin (Pitocin) infusion be discontinued immediately? a. A woman in early labor with contractions every 5 minutes lasting 40 seconds each b. A woman in active labor with contractions every 3 minutes lasting 60 seconds each c. A woman in active labor with contractions every 2 to 3 minutes lasting 70 to 80 seconds each d. A woman in transition with contractions every 1 to 2 minutes lasting 96 seconds each

d. A woman in transition with contractions every 1 to 2 minutes lasting 96 seconds each

To teach patients about the process of labor adequately, the nurse knows that which event is the best indicator of true labor? a. uterine contractions every 7 minutes b. Bloody show c. Fetal descent into the pelvic inlet d. Cervical dilation and effacement

d. Cervical dilation and effacement

What type of sexual abuse is this? Children in sex acts for profit, usually with changing partners. a. Child pornography b. Incest c. Exhibitionism d. Child prostitution e. Molestation f. Pedophilia

d. Child prostitution

Trauma to which site can result in a growth problem for children's long bones? a. Matrix b. Connective tissue c. Calcified cartilage d. Epiphyseal cartilage plate

d. Epiphyseal cartilage plate

Which is the priority concern in developing a teaching plan for the parents of a 15-month-old child? a. Toilet training guidelines b. Guidelines for weaning children from bottles c. Instructions on preschool readiness d. Instructions on a home safety assessment

d. Instructions on a home safety assessment A Although it is appropriate to give parents of a 15-month-old child toilet training guidelines, the child is not usually ready for toilet training, so it is not the priority teaching intervention. B Parents of a 15-month-old child should have been advised to beginning weaning from the breast or bottle at 6 to 12 months of age. C Educating a parent about preschool readiness is important and can occur later in the parents' educational process. The priority teaching intervention for the parents of a 15-month-old child is the importance of a safe environment. D Accidents are the major cause of death in children, including deaths caused by ingestion of poisonous materials. Home and environmental safety assessments are priorities in this age-group because of toddlers' increased motor skills and independence, which puts them at greater risk in an unsafe environment.

The Vietnamese mother of a child being seen in the clinic avoids eye contact with the nurse. Considering cultural differences, the best explanation for this is that the parent: a. Feels responsible for her child's illness. b. Feels inferior to nurse. c. Is embarrassed to seek health care. d. Is showing respect for nurse.

d. Is showing respect for nurse. In some ethnic groups eye contact is avoided. In the Vietnamese culture an individual may not look directly into the nurse's eyes as a sign of respect. The nurse providing culturally competent care would recognize that the mother does not feel responsible for her child's illness, she does not feel inferior to the nurse, and she is not embarrassed to seek health care as reasons for the mother to avoid eye contact with the nurse.

Anorexia nervosa may best be described as which of the following? a. Occurs most frequently in adolescent males b. Occurs most frequently in adolescents from lower socioeconomic groups c. Results from a posterior pituitary disorder d. Results in severe weight loss in the absence of obvious physical causes

d. Results in severe weight loss in the absence of obvious physical causes The etiology of anorexia remains unclear, but a distinct psychologic component is present. The diagnosis is based primarily on psychologic and behavioral criteria.

Which factor is most important in predisposing toddlers to frequent infections? a. Respirations are abdominal. b. Pulse and respiratory rates are slower than those in infancy. c. Defense mechanisms are less efficient than those during infancy. d. Toddlers have a short, straight internal ear canal and large lymph tissue.

d. Toddlers have a short, straight internal ear canal and large lymph tissue. Toddlers continue to have the short, straight internal ear canal of infants. The lymphoid tissue of the tonsils and adenoids continues to be relatively large. These two anatomic conditions combine to predispose the toddler to frequent infections. The abdominal respirations and lowered pulse and respiratory rate of toddlers do not affect their susceptibility to infection. The defense mechanisms are more efficient compared with those of infancy

When introducing solid foods to infants the nurse must educate parents that: a. Snacks should be foods that are liked by the infant b. finger foods allow for the development of the palmar grasp c.It is best to give all kinds of foods as long as the infant is 6 months of age d. fruits and vegetables can be steamed or boiled and then blended

d. fruits and vegetables can be steamed or boiled and then blended

A child who is overweight comes to the nurse's office 2 -3 times a week not feeling well. the nurse knows that psychosocial difficulties manifest in different ways. what question would be important for the nurse to ask? a. how much physical activity do you have during the day? b. who packs your lunch? c. Do you know about chronic diseases? d. what activities do you like doing with friends?

d. what activities do you like doing with friends?

What is helpful to tell a mother who is concerned about preventing sleep problems in her 2-year-old child? a. Have the child always sleep in a quiet, darkened room. b. Provide high-carbohydrate snacks before bedtime. c. Communicate with the child's daytime caretaker about eliminating the afternoon nap. d. Use a nightlight in the child's room.

d.Use a nightlight in the child's room. Feedback A A dark, quiet room may be scary to a preschooler. B High-carbohydrate snacks increase energy and do not promote relaxation. C Most 2-year-olds take one nap each day. Many give up the habit by age 3 years. Insufficient rest during the day can lead to irritability and difficulty sleeping at night. D The preschooler has a great imagination. Sounds and shadows can have a negative effect on sleeping behavior. Nightlights provide the child with the ability to visualize the environment and decrease the fear felt in a dark room.

What type of sexual abuse is this? Touching, founding, kissing, masturbation, oral sexual acts. a. Child pornography b. Incest c. Exhibitionism d. Child prostitution e. Molestation f. Pedophilia

e. Molestation

What type of sexual abuse is this? "love of child" preference of a child over an adult sexually involving excitement". a. Child pornography b. Incest c. Exhibitionism d. Child prostitution e. Molestation f. Pedophilia

f. Pedophilia


Kaugnay na mga set ng pag-aaral

Bio 1108 Unit 1 Learning Curve Questions

View Set

which bone forms the bridge of the nose

View Set

intro to business chapter 7: Management

View Set

AP US History: Unit's 1-3 (Give Me Liberty)

View Set